Thomas' Calculus 13th Edition

Published by Pearson
ISBN 10: 0-32187-896-5
ISBN 13: 978-0-32187-896-0

Chapter 10: Infinite Sequences and Series - Section 10.2 - Infinite Series - Exercises 10.2 - Page 580: 41

Answer

converges to $1$.

Work Step by Step

consider $a_n=\dfrac{4}{(4n-3)(4n+1)}=\dfrac{1}{4n-3}-\dfrac{1}{4n+1}$ The nth partial sums are: $ s_n=(1-\dfrac{1}{5})+(\dfrac{1}{5}-\dfrac{1}{9})+....(\dfrac{1}{4n-3}-\dfrac{1}{4n+1})=(1-\dfrac{1}{4n+1})$ Now, apply limits $\lim\limits_{n \to \infty} s_n=(1)-\lim\limits_{n \to \infty} \dfrac{1}{4n+1}=(1)+(0)=1$ Thus, As $n \to \infty$ then $s_n \to 1$ . Hence, the series converges to $1$.
Update this answer!

You can help us out by revising, improving and updating this answer.

Update this answer

After you claim an answer you’ll have 24 hours to send in a draft. An editor will review the submission and either publish your submission or provide feedback.